반응형

[해석학] 10. 미분



\(x_{0}\)의 근방에서 정의된 함수 \(f\)에 대하여 임의의 \(\epsilon>0\)에 대해 \(L\in\mathbb{R}\)과 \(\delta>0\)이 존재해서 \(0<|x-x_{0}|<\delta\)일 때 \(\displaystyle\left|\frac{f(x)-f(x_{0})}{x-x_{0}}-L\right|<\epsilon\)이면, \(f\)는 \(x=x_{0}\)에서 미분가능(differentiable)하다고 하고, \(L\)을 \(x=x_{0}\)에서의 미분계수(derivative of \(f\) at \(x_{0}\))라고 하며 \(f'(x_{0})\)으로 나타낸다.

이 미분계수의 정의는 미적분학에서의 미분계수의 정의 \(\displaystyle f'(x_{0})=\lim_{x\,\rightarrow\,x_{0}}{\frac{f(x)-f(x_{0})}{x-x_{0}}}\)을 \(\epsilon-\delta\)를 이용하여 나타낸 것이다.

\(f\)가 모든 \(x\in[a,\,b]\)에서 미분가능하면 \(f\)는 \([a,\,b]\)에서 미분가능하다고 한다. \(x=x_{0}\)에서 \(f\)의 미분계수가 존재하지 않으면, \(f\)는 \(x=x_{0}\)에서 미분가능하지 않다고 한다.

\(\displaystyle f_{-}'(x_{0})=\lim_{x\,\rightarrow\,x_{0}-}{\frac{f(x)-f(x_{0})}{x-x_{0}}}\)의 값이 존재할 때, 이 값을 \(x=x_{0}\)에서 \(f\)의 좌미분계수(left-side derivative of \(f\) at \(x_{0}\)), \(\displaystyle f_{+}'(x_{0})=\lim_{x\,\rightarrow\,x_{0}+}{\frac{f(x)-f(x_{0})}{x-x_{0}}}\)의 값이 존재할 때, 이 값을 \(x=x_{0}\)에서 \(f\)의 우미분계수(right-side derivative of \(f\) at \(x_{0}\))라고 한다.

따라서 \(f\)가 \(x_{0}\in(a,\,b)\)에서 미분가능할 필요충분조건은 \(f_{-}'(x_{0})\)와 \(f_{+}'(x_{0})\)가 모두 존재하고 \(f_{-}'(x_{0})=f_{+}'(x_{0})\)이다.

다음의 함수$$f(x)=\begin{cases}x^{2}\sin\frac{1}{x},\,(x\neq0)\\0,\,(x=0)\end{cases},\,g(x)=\begin{cases}x\sin\frac{1}{x},\,(x\neq0)\\0,\,(x=0)\end{cases}$$에 대하여$$\frac{f(x)-f(0)}{x-0}=x\sin\frac{1}{x},\,\frac{g(x)-g(0)}{x-0}=\sin\frac{1}{x}$$이고 \(\displaystyle\lim_{x\,\rightarrow\,0}{x\sin\frac{1}{x}}=0\)이므로 \(f\)는 \(x=0\)에서 미분가능하고 \(f'(0)=0\)이나 \(\displaystyle\lim_{x\,\rightarrow\,0}{\sin\frac{1}{x}}\)의 값은 존재하지 않으므로 \(g\)는 \(x=0\)에서 미분가능하지 않다.


함수 \(f\)가 \(x=a\)에서 미분가능하면, \(f\)는 \(x=a\)에서 연속이다. 

증명: \(f\)가 \(x=a\)에서 미분가능하므로 \(\delta_{1}>0\)가 존재해서 \(0<|x-a|<\delta_{1}\)일 때 \(\displaystyle\left|\frac{f(x)-f(a)}{x-a}-f'(a)\right|<1\)이고 \(|f(x)-f(a)|<|x-a|(1+|f'(a)|)\)이다. 임의의 \(\epsilon>0\)에 대해 \(\displaystyle\delta=\min\left\{1,\,\frac{\epsilon}{1+|f'(a)|}\right\}\)라고 하면 \(|x-a|<\delta\)일 때$$|f(x)-f(a)|<|x-c|(1+|f'(a)|)<(1+|f'(a)|)\frac{\epsilon}{1+|f'(a)|}=\epsilon$$이다.

이 정리의 역은 성립하지 않는다. 함수 \(f(x)=|x|\)는 \(x=0\)에서 연속이고 \(x=0\)에서의 미분계수를 구하면$$\begin{align*}f_{-}'(0)&=\lim_{x\,\rightarrow\,0-}{\frac{f(x)-f(0)}{x-0}}=\lim_{x\,\rightarrow\,0-}{\frac{-x}{x}}=-1\\f_{+}'(0)&=\lim_{x\,\rightarrow\,0+}{\frac{f(x)-f(0)}{x-0}}=\lim_{x\,\rightarrow\,0+}{\frac{x}{x}}=1\end{align*}$$이므로 \(f_{-}'(0)\neq f_{+}'(0)\)이고 따라서 \(x=0\)에서 미분가능하지 않다.


\(D\)를 \(f\)가 미분가능한 점들의 집합이라고 하자. \(\displaystyle f'(x)=\lim_{h\,\rightarrow\,0}{\frac{f(x+h)-f(x)}{h}}\)로 정의되는 함수 \(f:\,D\,\rightarrow\,\mathbb{R}\)을 \(f\)의 도함수(derivative)라고 하고, \(f'\) 또는 \(\displaystyle\frac{dy}{dx},\,\frac{df(x)}{dx}\)로 나타낸다.


함수 \(f\)가 \(x=a\)에서 미분가능할 필요충분조건은 \(L\in\mathbb{R}\)과 함수 \(\eta_{f}\)가 존재해서 \(\displaystyle\lim_{x\,\rightarrow\,a}{\eta_{f}}=0\)이고, \(x\neq a\)일 때$$f(x)=f(a)+L(x-a)+\eta_{f}(x)(x-c)$$이며 이때 \(L=f'(a)\)이다.

증명:

(\(\Rightarrow\)): \(\displaystyle f'(a)=\lim_{x\,\rightarrow\,a}{\frac{f(x)-f(a)}{x-a}}\)이고 \(\eta_{f}(x)=\frac{f(x)-f(a)}{x-a}-f'(a)\)라고 하면, \(\displaystyle\lim_{x\,\rightarrow\,a}{\eta_{f}(x)}=0\)이고, \(x\neq a\)일 때$$f(x)=f(c)+L(x-c)+\eta_{f}(x)(x-a)$$이다.  

(\(\Leftarrow\)): \(\displaystyle\frac{f(x)-f(a)}{x-a}=L+\eta_{f}(x)\)이므로 \(\displaystyle\lim_{x\,\rightarrow\,a}{\frac{f(x)-f(a)}{x-a}}=L\)이고 따라서 \(x=a\)에서 미분가능하고 \(L=f'(a)\)이다.

이 정리는 연쇄법칙의 증명에 이용되는 정리이다.


미분가능한 함수 \(f,\,g\)에 대하여

(1) \((f+g)'(x)=f'(x)+g'(x)\) 

(2) \((kf)'(x)=kf'(x)\,(k\in\mathbb{R})\) 

(3) \((fg)'(x)=f'(x)g(x)+f(x)g'(x)\) 

(4) \(\displaystyle\left(\frac{f}{g}\right)'(x)=\frac{f'(x)g(x)-f(x)g'(x)}{\{g(x)\}^{2}}\,(g(x)\neq0)\) 

(1)과 (2)의 증명은 간단하므로 생략하고, (3)과 (4)의 증명은 다음의 등식을 이용한다.

(3): \(\displaystyle\frac{f(x)g(x)-f(x_{0})g(x_{0})}{x-x_{0}}=f(x)\frac{g(x)-g(x_{0})}{x-x_{0}}+g(x_{0})\frac{f(x)-f(x_{0})}{x-x_{0}}\)

(4): \(\displaystyle\frac{1}{x-x_{0}}\left(\frac{f(x)}{g(x)}-\frac{f(x_{0})}{g(x_{0})}\right)=\frac{1}{g(x)g(x_{0})}\left\{g(x_{0})\frac{f(x)-f(x_{0})}{x-x_{0}}-f(x_{0})\frac{g(x)-g(x_{0})}{x-x_{0}}\right\}\) 


연쇄법칙(chain rule)

함수 \(f\)가 \(x=x_{0}\)에서 미분가능하고, 함수 \(g\)가 \(y=f(x_{0})\)에서 미분가능하면, 합성함수 \((g\circ f)(x)\)는 \(x=x_{0}\)에서 미분가능하고 \((g\circ f)'(x_{0})=f'(x_{0})g'(f(x_{0}))\)이다.

증명: \(y_{0}=f(x_{0})\)라고 하자. \(f\)와 \(g\)가 각각 \(x=x_{0}\), \(y=y_{0}\)에서 미분가능하므로 \(\eta_{f}\), \(\eta_{g}\)가 존재해서 \(\displaystyle\lim_{x\,\rightarrow\,x_{0}}{\eta_{f}(x)}=0\), \(\displaystyle\lim_{y\,\rightarrow\,y_{0}}{\eta_{g}(y)}=0\)이고$$\begin{align*}f(x)-f(x_{0})&=(x-c)\{f'(x_{0})+\eta_{f}(x)\}\\g(y)-g(y_{0})&=(y-y_{0})\{g'(y_{0})+\eta_{g}(y)\}\end{align*}$$이므로$$\begin{align*}(f\circ g)(x)-(f\circ g)(x_{0})&=g(f(x))-g(f(x_{0}))\\&=\{f(x)-f(x_{0})\}\{g'(y_{0})+\eta_{g}(y)\}\\&=(x-x_{0})\{f'(x_{0})+\eta_{f}(x)\}\{g'(y_{0})+\eta_{g}(y)\}\end{align*}$$이다. 따라서$$(f\circ g)'(x_{0})=\lim_{x\,\rightarrow\,x_{0}}{\frac{(g\circ f)(x)-(g\circ f)(x_{0})}{x-x_{0}}}=f'(x_{0})g'(y_{0})=f'(x_{0})g'(f(x_{0}))$$이다. 연쇄법칙으로부터 합성함수 \(y=(g\circ f)\)의 도함수가 \((g\circ f)'(x)=f'(x)g(f(x))\)임을 알 수 있다.


역함수가 존재하는 함수 \(f\)에 대해 \(f'(x)\neq0\)이면, \(\displaystyle\frac{dx}{dy}\)가 존재하고$$(f^{-1})'(y)=\frac{1}{f'(f^{-1}(y))}$$이다.

증명: 함수 \(y=f(x)\)의 역함수가 존재하므로 \(x=f^{-1}(y)\)이고 \(y_{0}=f(x_{0})\)라고 하면$$(f^{-1})'(y)=\lim_{y\,\rightarrow\,y_{0}}{\frac{f^{-1}(y)-f^{-1}(y_{0})}{y-y_{0}}}=\lim_{x\,\rightarrow\,x_{0}}{\frac{x-x_{0}}{f(x)-f(x_{0})}}=\lim_{x\,\rightarrow\,x_{0}}{\frac{1}{\frac{f(x)-f(x_{0})}{x-x_{0}}}}=\frac{1}{f'(x_{0})}=\frac{1}{f'(f^{-1}(y_{0}))}$$이고 따라서 \(\displaystyle(f^{-1})'(y)=\frac{1}{f'(f^{-1}(y))}\)이다.


함수 \(f\)에 대해 

(1) \(f(x)\leq f(x_{0})\)이면, \(f(x)\)는 \(x=x_{0}\)에서 최대(maximum)이고, \(f(x_{0})\)는 최댓값(maximum)이다. 

(2) \(f(x_{0})\leq f(x)\)이면, \(f(x)\)는 \(x=x_{0}\)에서 최소(minimum)이고, \(f(x_{0})\)는 최솟값(minimum)이다. 

(3) \(\delta>0\)가 존재해서 \(x\in(x_{0}-\delta,\,x_{0}+\delta)\)에 대해 \(f(x)\leq f(x_{0})\)이면, \(f\)는 \(x=x_{0}\)에서 극대(local maximum)이고, \(f(x_{0})\)는 극댓값(local maximum value)이다. 

(4) \(\delta>0\)가 존재해서 \(x\in(x_{0}-\delta,\,x_{0}+\delta)\)에 대해 \(f(x_{0})\leq f(x)\)이면, \(f\)는 \(x=x_{0}\)에서 극소(local minimum)이고, \(f(x_{0})\)는 극솟값(local minimum value)이다. 

극댓값과 극솟값을 통틀어 극값(extremum value)이라고 한다.


페르마 정리(Fermat's theorem)

함수 \(f\)가 \(x=x_{0}\)에서 극값을 갖고 미분가능하면, \(f'(x_{0})=0\)이다.

증명:

(i) \(f\)가 \(x=x_{0}\)에서 극대라고 하자. \(h>0\)일 때 \(\displaystyle f_{+}'(x_{0})=\lim_{x\,\rightarrow\,x_{0}+}{\frac{f(x)-f(x_{0})}{x-x_{0}}}\leq0\)이고, \(h<0\)일 때 \(\displaystyle f_{-}'(x_{0})=\lim_{x\,\rightarrow\,x_{0}-}{\frac{f(x)-f(x_{0})}{x-x_{0}}}\geq0\)이다.

\(f\)가 \(x=x_{0}\)에서 미분가능하므로 \(f_{+}'(x_{0})=f'(x_{0})=f_{-}'(x_{0})\)이고, \(f_{+}'(x_{0})\leq0\), \(f_{-}'(x_{0})\geq0\)이므로 \(f'(x_{0})=0\)이다. 

(ii) \(f\)가 \(x=x_{0}\)에서 극소라고 하자. \(h>0\)일 때 \(\displaystyle f_{+}'(x_{0})=\lim_{x\,\rightarrow\,x_{0}+}{\frac{f(x)-f(x_{0})}{x-x_{0}}}\geq0\)이고, \(h<0\)일 떄 \(\displaystyle f_{-}'(x_{0})=\lim_{x\,\rightarrow\,x_{0}}{\frac{f(x)-f(x_{0})}{x-x_{0}}}\leq0\)이다.

\(f\)가 \(x=x_{0}\)에서 미분가능하므로 \(f_{+}'(x_{0})=f'(x_{0})=f_{-}'(x_{0})\)이고, \(f_{+}'(x_{0})\geq0\), \(f_{-}'(x_{0})\leq0\)이므로 \(f'(x_{0})=0\)이다.

이 정리의 역은 성립하지 않는다. 함수 \(f(x)=x^{3}\)에 대해서 \(f'(0)=0\)이나 \(x=0\)에서 극값을 갖지 않는다.

참고로 미분가능하지 않은 점에서 극값을 가질 수 있다. 함수 \(f(x)=|x|\)는 \(x=0\)에서 미분가능하지 않으나 극소이다.

* \(f'(x_{0})=0\)이거나 \(f'(x_{0})\)이 존재(미분가능)하지 않으면, \(x_{0}\)를 \(f\)의 임계점(critical point)이라고 한다.


롤의 정리(Rolle's theorem)

함수 \(f\)가 \([a,\,b]\)에서 연속이고 \((a,\,b)\)에서 미분가능하며 \(f(a)=f(b)\)이면, \(c\in(a,\,b)\)가 존재해서 \(f'(c)=0\)이다. 

증명: \(f\)가 \([a,\,b]\)에서 연속이므로 최댓값최솟값 정리로부터 최댓값 \(f(x_{M})\)과 최솟값 \(f(x_{m})\)을 갖는다.

(i) \(f(x_{m})=f(x_{M})\)(상수함수)일 때, \(f\)는 상수함수이므로 모든 \(x\in[a,\,b]\)에 대하여 \(f'(x)=0\)이다.  

(ii) \(f(x_{m})<f(x_{M})\)일 때, \(f\)는 상수함수가 아니므로 \(x_{m}\in(a,\,b)\) 또는 \(x_{M}\in(a,\,b)\)이고 \(f\)는 \(x=x_{M}\)에서 극대이거나 \(x=x_{m}\)에서 극소이다. 즉 \(f'(x_{M})=0\) 또는 \(f'(x_{m})=0\)이고 따라서 \(c=x_{M}\) 또는 \(c=x_{m}\)이다.  


미분에 대한 평균값 정리(mean-value theorem for differential calculus)   

함수 \(f\)가 \([a,\,b]\)에서 연속이고 \((a,\,b)\)에서 미분가능하면, \(c\in(a,\,b)\)가 존재해서 \(\displaystyle f'(c)=\frac{f(b)-f(a)}{b-a}\)이다. 

증명: \(\displaystyle F(x)=f(x)-\frac{f(b)-f(a)}{b-a}(x-a)-f(a)\)라고 하자. \(F(a)=F(b)=0\)이므로 롤의 정리에 의해 \(c\in(a,\,b)\)가 존재해서 \(F'(c)=0\)이다. \(\displaystyle F'(c)=f'(c)-\frac{f(b)-f(a)}{b-a}=0\)이므로 \(\displaystyle f'(c)=\frac{f(b)-f(a)}{b-a}\)이다.   


코시의 평균값 정리(Cauchy mean-value theorem)

함수 \(f,\,g\)가 \([a,\,b]\)에서 연속이고 \((a,\,b)\)에서 미분가능하면 \(c\in(a,\,b)\)가 존재해서 \(f'(c)\{g(b)-g(a)\}=g'(c)\{f(b)-f(a)\}\)이다.

증명: \(F(x)=f(x)\{g(b)-g(a)\}-g(x)\{f(b)-f(a)\}\)라고 하자. \(F\)는 \([a,\,b]\)에서 연속이고 \((a,\,b)\)에서 미분가능하며$$\begin{align*}F(a)&=f(a)\{g(b)-g(a)\}-g(a)\{f(b)-f(a)\}=f(a)g(b)-g(a)f(b)\\F(b)&=f(b)\{g(b)-g(a)\}-g(b)\{f(b)-f(a)\}=f(a)g(b)-g(a)f(b)\end{align*}$$이므로 롤의 정리에 의해 \(c\in(a,\,b)\)가 존재해서 \(F'(c)=0\)이다. \(\displaystyle F'(c)=f'(c)\{g(b)-g(a)\}-g'(c)\{f(b)-f(a)\}=0\)이므로 \(f'(c)\{g(b)-g(a)\}=g'(c)\{f(b)-f(a)\}\)이다.


함수 \(f\)가 \([a,\,b]\)에서 연속이고 모든 \(x\in(a,\,b)\)에 대하여   

(1) \(f'(x)=0\)이면, \(f\)는 \([a,\,b]\)에서 상수함수이다. 

(2) \(f'(x)\geq0\)이면, \(f\)는 \([a,\,b]\)에서 단조증가이다. 

(3) \(f'(x)\leq0\)이면, \(f\)는 \([a,\,b]\)에서 단조감소이다.  

증명: \(x_{1},\,x_{2}\in[a,\,b]\,(x_{1}<x_{2})\)라고 하자. 평균값 정리에 의해 \(c\in(x_{1},\,x_{2})\)가 존재해서 \(\displaystyle\frac{f(x_{2})-f(x_{1})}{x_{2}-x_{1}}=f'(c)\)이다.

(1) \(\displaystyle f'(c)=\frac{f(x_{2})-f(x_{1})}{x_{2}-x_{1}}=0\)이므로 \(f(x_{1})=f(x_{2})\)이고 따라서 \(f\)는 \([a,\,b]\)에서 상수함수이다. 

(2) \(\displaystyle f'(c)=\frac{f(x_{2})-f(x_{1})}{x_{2}-x_{1}}\geq0\)이므로 \(f(x_{1})\leq f(x_{2})\)이고 따라서 \(f\)는 \([a,\,b]\)에서 단조증가이다. 

(3) \(\displaystyle f'(c)=\frac{f(x_{2})-f(x_{1})}{x_{2}-x_{1}}\leq0\)이므로 \(f(x_{1})\geq f(x_{2})\)이고 따라서 \(f\)는 \([a,\,b]\)에서 단조감소이다.


구간 \(I\)에서 정의된 함수 \(f\)에 대하여 임의의 \(x,\,y\in I\)에 대해 \(M>0\)이 존재해서 \(|f(x)-f(y)|\leq M|x-y|\)이면, \(f\)는 립쉬츠 조건(Lipschitz condition)을 만족한다고 한다.

*구간 \(I\)에서 정의된 함수 \(f\)에 대하여 \(f'\)가 \(I\)에서 유계이면, 모든 \(x\in I\)에 대하여 \(M>0\)이 존재해서 \(|f(x)|\leq M\)이고, 평균값 정리에 의해 임의의 \(x,\,y\in I\,(x<y)\)에 대해 \(c\in(x,\,y)\)가 존재해서 \(\displaystyle\frac{f(y)-f(x)}{y-x}=f'(c)\)이므로 따라서 \(|f(x)-f(y)|\leq M|x-y|\)이다.


로피탈 법칙(L'Hospital's rule)

(1) \(f',\,g'\)이 \(x=a\)의 제거된 근방에서 존재하고 \(g(x)\neq0\), \(f(a)=g(a)=0\), \(\displaystyle\lim_{x\,\rightarrow\,a+}{\frac{f'(x)}{g'(x)}}=L\)이면, \(\displaystyle\lim_{x\,\rightarrow\,a+}{\frac{f(x)}{g(x)}}=\lim_{x\,\rightarrow\,a+}{\frac{f'(x)}{g'(x)}}=L\)이다. 

(2) \([a,\,\infty)\)에서 \(f',\,g'\)이 존재하고 \(g'(x)\neq0\), \(\displaystyle\lim_{x\,\rightarrow\,\infty}{f(x)}=\lim_{x\,\rightarrow\,\infty}{g(x)}=0\), \(\displaystyle\lim_{x\,\rightarrow\,\infty}{\frac{f'(x)}{g'(x)}}=L\)이면, \(\displaystyle\lim_{x\,\rightarrow\,\infty}{\frac{f(x)}{g(x)}}=\lim_{x\,\rightarrow\,\infty}{\frac{f'(x)}{g'(x)}}=L\)이다.  

(3) \((a,\,a+\delta)\,(\delta>0)\)에서 \(f',\,g'\)이 존재하고 \(\displaystyle\lim_{x\,\rightarrow\,a+}{f(x)}=\lim_{x\,\rightarrow\,a+}{g(x)}=\infty\), \(g(x)\neq0\), \(g'(x)\neq0\), \(\displaystyle\lim_{x\,\rightarrow\,a+}{\frac{f'(x)}{g'(x)}}=L\)이면, \(\displaystyle\lim_{x\,\rightarrow\,a+}{\frac{f(x)}{g(x)}}=\lim_{x\,\rightarrow\,a+}{\frac{f'(x)}{g'(x)}}=L\)이다.   

증명:

(1) \(\displaystyle\lim_{x\,\rightarrow\,a}{\frac{f'(x)}{g'(x)}}=L\)이므로 임의의 \(\epsilon>0\)에 대해 \(\delta>0\)이 존재해서 \(a<x<a+\delta\)일 때 \(\displaystyle\left|\frac{f'(x)}{g'(x)}-L\right|<\epsilon\)이고, 코시의 평균값 정리에 의해 임의의 \(x\in(a,\,a+\delta)\)에 대해 \(c_{x}\in(a,\,x)\)가 존재해서$$\frac{f'(c_{x})}{g'(c_{x})}=\frac{f(x)-f(a)}{g(x)-g(a)}=\frac{f(x)}{g(x)}$$이다. 따라서 \(a<x<a+\delta\)일 때$$\displaystyle\left|\frac{f(x)}{g(x)}-L\right|=\left|\frac{f'(x)}{g'(x)}-L\right|<\epsilon$$이고 \(\displaystyle\lim_{x\,\rightarrow\,a+}{\frac{f(x)}{g(x)}}=L\)이다.   

(2) \(\displaystyle t=\frac{1}{x}\)라고 하면 \(x\,\rightarrow\,\infty\)일 때 \(t\,\rightarrow\,0+\)이므로 (1)에 의해 성립한다.  

(3) \(\displaystyle t=\frac{1}{x-a}\)라고 하면 \(x\,\rightarrow\,a+\)일 때 \(t\,\rightarrow\,\infty\)이므로 (2)에 의해 성립한다.

*이 정리에서 (1)은 좌극한, 양쪽극한인 경우에도 성립하고, (2)는 음의 무한대로 갈 때도 성립하며, (3) 또한 좌극한, 양쪽극한, 음의 무한대인 경우에도 성립한다.


함수 \(f\)의 도함수 \(f'\)이 미분가능하면 그 도함수는 \(\displaystyle f''(x)=\lim_{h\,\rightarrow\,\infty}{\frac{f'(x+h)-f'(x)}{h}}\)이고, 이 도함수를 \(f\)의 2계도함수(second derivative)라고 한다. 일반적으로 \(f\)의 \(n-1\)계도함수 \(f^{(n-1)}\)가 미분가능하면 그 도함수를 \(f^{(n-1)}\)의 \(n\)계도함수(\(n\)-th derivative)라고 하고 \(f^{(n-1)}\)로 나타낸다.

집합 \(I\)에서 정의된 함수 \(f\)가 \(n\)계도함수 \(f^{(n)}\)을 갖고 \(f^{(n)}\)이 연속이면, \(f\)를 \(C^{(n)}\)급 함수(function of class \(C^{(n)}\))라고 하고, 이러한 함수 전체의 집합을 \(C^{(n)}(I)\)로 나타내며, 모든 \(n\in\mathbb{N}\)에 대해 \(f^{(n)}\)이 존재하고, \(f^{(n)}\)이 연속이면, \(f\)를 \(C^{(\infty)}\)급 함수(function of class \(C^{(\infty)}\))라 하고, 이러한 함수 전체의 집합을 \(C^{(\infty)}(I)\)로 나타낸다. 

*\(n\)이 임의의 자연수일때 \(f,\,g\in C^{n}(I)\)에 대하여$$(f+g)(x)=f(x)+g(x),\,(kf)(x)=kf(x)\,(k\in\mathbb{R})$$이고, \(\displaystyle C^{(\infty)}=\bigcap_{n=1}^{\infty}{C^{(n)}(I)}\)이다.     


함수 \(f\in C^{(n)}([a,\,b])\)에 대하여 \(\displaystyle P_{n,\,f}(x)=\sum_{k=0}^{n}{\frac{f^{(k)}(x_{0})}{n!}(x-x_{0})^{n}}\)을 \(x=x_{0}\)에서 \(f\)의 \(n\)차 테일러 다항식(Taylor polynomial function)이라고 하고, \(R_{n}=f-P_{n,\,f}\)를 \(x=x_{0}\)에서 \(f\)의 나머지(remainder)라고 한다.


테일러 정리(Taylor theorem)

\(f\in C^{(n+1)}([a,\,b])\)라고 하자. 모든 \(x\in(a,\,b)\)에 대하여 \(\displaystyle f(x)=\sum_{k=0}^{n}{\frac{f^{(k)}(x_{0})}{k!}}(x-x_{0})^{k}+R_{n}(x)\)이고, \(R_{n}\)은 \(\displaystyle R_{n}(x)=\frac{f^{(n+1)}(c)}{(n+1)!}(x-x_{0})^{n+1}\,(x_{0}<c<x)\)이다.

증명: \(x_{0}<t<x\)인 \(t\)에 대하여 함수 \(F\)를 \(\displaystyle F(t)=f(x)-\sum_{k=0}^{n}{\frac{f^{(n)}(t)}{n!}(x-t)^{n}}\)로 정의하자. 그러면 \(\displaystyle F'(t)=-\frac{(x-t)^{n}}{n!}f^{(n+1)}(t)\)이다. \(\displaystyle G(t)=F(t)-\left(\frac{x-t}{x-x_{0}}\right)^{n+1}F(x_{0})\)라고 하면, \(G\)는 미분가능하고 \(G(x_{0})=F(x_{0})=-F(x_{0})=0\), \(G(x)=F(x)=0\)이므로 롤의 정리에 의해 \(c\in(x,\,x_{0})\)가 존재해서$$G'(c)=F'(c)+(n+1)\frac{(x-c)^{n}}{(x-x_{0})^{n+1}}F(x_{0})=0$$이고 따라서$$F(x_{0})=-\frac{1}{n+1}\frac{(x-x_{0})^{n+1}}{(x-c)^{n}}F'(c)=\frac{f^{(n+1)}(c)}{(n+1)!}(x-x_{0})^{n+1}=R_{n}(x)$$이다.


함수 \(f\)가 \(f\in C^{(\infty)}(I)\)이고 \(a\in I\)라고 하자. 모든 \(x\in I\)에 대하여 다음의 등식$$f(x)=\sum_{n=0}^{\infty}{\frac{f^{(n)}(a)}{n!}(x-a)^{n}}$$이 성립할 필요충분조건은 모든 \(x\in I\)에 대하여 \(\displaystyle\lim_{n\,\rightarrow\,\infty}{R_{n}(x)}=0\)이다.

증명: \(\displaystyle S_{n}(x)=\sum_{k=0}^{n}{\frac{f^{(k)}(a)}{k!}(x-a)^{k}}\)라고 하면 테일러 정리에 의해 \(S_{n}=f(x)-R_{n}(x)\)이고 따라서 \(\displaystyle f(x)=\lim_{n\,\rightarrow\,\infty}{S_{n}(x)}=\sum_{n=0}^{\infty}{\frac{f^{(n)}(a)}{n!}(x-a)^{n}}\)일 필요충분조건은 \(\displaystyle\lim_{n\,\rightarrow\,\infty}{R_{n}(x)}=0\)이다.

이 정리에서의 급수 \(\displaystyle f(x)=\sum_{n=0}^{\infty}{\frac{f^{(n)}(a)}{n!}(x-a)^{n}}\)를 테일러 급수(Taylor series)라고 하고, \(a=0\)일 때의 급수 \(\displaystyle f(x)=\sum_{n=0}^{\infty}{\frac{f^{(n)}(0)}{n!}}x^{n}\)을 매클로린 급수(Maclaurin series)라고 한다.     

함수 \(f\)가 \(f\in C^{(\infty)}(I)\)이고 \(a\in I\)에서의 테일러 급수가 \(I\)에서 \(f\)에 수렴하면 \(f\)는 \(x=a\)에서 해석적(analytic)이라고 한다.

다음과 같이 정의된 함수$$f(x)=\begin{cases}e^{-\frac{1}{x^{2}}},\,(x\neq0)\\0,\,(x=0)\end{cases}$$는 \(x\in\mathbb{R}-\{0\}\)에서 \(f(x)\neq0\)이고, \(f\in C^{(\infty)}(\mathbb{R})\)이며 모든 자연수 \(n\)에 대해 \(f^{(n)}(0)=0\)이므로 \(f\)의 \(x=0\)에서의 테일러 급수는 \(f\)로 수렴하지 않는다. 따라서 이 함수 \(f\)는 해석적이지 않다.


\([a,\,b]\)에서 정의된 미분가능한 함수열 \(\{f_{n}\}\)이 다음의 두 조건

(i) 어떤 \(x_{0}\in[a,\,b]\)에서 \(\{f_{n}(x_{0})\}\)가 수렴한다.

(ii) 함수열 \(\{f_{n}'\}\)이 \([a,\,b]\)에서 어떤 함수에 균등수렴한다.

를 만족하면, 다음이 성립한다.

(1) \(\{f_{n}\}\)은 \([a,\,b]\)에서 적당한 함수 \(f\)에 균등수렴한다.

(2) \(f\)는 \([a,\,b]\)에서 미분가능하고, 임의의 \(x\in[a,\,b]\)에 대해 \(\displaystyle\lim_{n\,\rightarrow\,\infty}{f_{n}'(x)}=f'(x)\)이다.              

증명:

(1) 조건 (i)에 의해 임의의 \(\epsilon>0\)에 대하여 \(K\in\mathbb{N}\)가 존재해서 임의의 \(n,\,m\geq K\), \(x\in[a,\,b]\)에 대해 \(\displaystyle\left|f_{n}(x_{0})-f_{m}(x_{0})\right|<\frac{\epsilon}{2}\)이고, \(\displaystyle|f_{n}'(x_{0})-f_{m}'(x_{0})|<\frac{\epsilon}{2(b-a)}\)이다. 따라서 평균값 정리로부터 임의의 \(x,\,t\in[a,\,b]\)에 대해 \(t_{x}\in(x,\,t)\)가 존재해서$$\begin{align*}|f_{n}(x)-f_{m}(x)-\{f_{n}(t)-f_{m}(t)\}|&=|\{f_{n}(x_{0})-f_{m}(x_{0})\}-\{f_{n}(t)-f_{m}(t)\}|\\&\leq|\{f_{n}'(t_{x})-f_{m}'(t_{x})\}(x-t)|\\&\leq\frac{\epsilon}{2(b-a)}|x-t|\\&<\frac{\epsilon}{2}\end{align*}$$이고$$\begin{align*}|f_{n}(x)-f_{m}(x)|&\leq|f_{n}(x)-f_{m}(x)-f_{n}(x_{0})+f_{m(x_{0})}|+|f_{n}(x_{0})-f_{m}(x_{0})|\\&<\frac{\epsilon}{2}+\frac{\epsilon}{2}=\epsilon\end{align*}$$이므로 \(\{f_{n}\}\)은 \([a,\,b]\)에서 균등수렴한다. 극한함수 \(f\)를 \(\displaystyle f(x)=\lim_{n\,\rightarrow\,\infty}{f_{n}(x)}\)로 정의하면, \(\{f_{n}\}\)은 \(f\)로 균등수렴한다.

(2) \(f\)가 \([a,\,b]\)에서 미분가능함을 보이자. \(x\in[a,\,b]\)를 고정하고 \(t\in[a,\,b](t\neq x)\)에 대하여$$h_{n}(t)=\frac{f_{n}(t)-f_{n}(x)}{t-x},\,h(t)=\frac{f(t)-f(x)}{t-x}$$라고 하자. 그러면 \(\displaystyle\lim_{t\,\rightarrow\,x}{h_{n}(t)}=f_{n}'(x)\)이고$$|h_{m}(t)-h_{n}(t)|=\left|\frac{f_{n}(x)-f_{m}(x)-f_{n}(t)+f_{m}(t)}{x-t}\right|\leq\frac{\epsilon}{2(b-a)}$$이므로 \(\{h_{n}\}\)은 \([a,\,b]-\{x\}\)에서 균등수렴하고, \(f_{n}\)이 \(f\)로 균등수렴하므로 \(h_{n}\)은 \([a,\,b]-\{x\}\)에서 \(h\)로 균등수렴한다. 따라서$$f'(x)=\lim_{t\,\rightarrow\,x}{h(t)}=\lim_{t\,\rightarrow\,x}{\lim_{n\,\rightarrow\,\infty}{h_{n}(t)}}=\lim_{n\,\rightarrow\,\infty}{\lim_{t\,\rightarrow\,x}{h_{n}(t)}}=\lim_{n\,\rightarrow\,\infty}{f_{n}'(t)}=f'(x)$$이다. 

이 정리로부터 다음의 정리를 얻는다.

\(f_{n}\)이 \([a,\,b]\)에서 미분가능하고, \(\displaystyle\sum_{n=1}^{\infty}{f_{n}}\)이 다음의 두 조건

(i) 어떤 \(x_{0}\in[a,\,b]\)에 대하여 급수 \(\displaystyle\sum_{n=0}^{\infty}{f_{n}(x_{0})}\)가 수렴한다.

(ii) \(\displaystyle\sum_{n=1}^{\infty}{f_{n}'}\)은 \([a,\,b]\)에서 균등수렴한다.

를 만족하면, 다음이 성립한다.

(1) \(\displaystyle\sum_{n=1}^{\infty}{f_{n}}\)은 \([a,\,b]\)에서 미분가능한 함수 \(f\)로 수렴한다. 

(2) \(f\)는 \([a,\,b]\)에서 미분가능하고, \([a,\,b]\)에서 \(\displaystyle\sum_{n=1}^{\infty}{f_{n}'}=f'\)이다.

이 정리의 증명은 함수열 \(\displaystyle S_{n}(x)=\sum_{k=1}^{n}{f_{k}(x)}\)를 앞 정리에 적용한다. 

함수열 \(\displaystyle f_{n}(x)=\sum_{k=0}^{n}{\frac{1}{2^{k}}\cos(3^{k}x)}\)는 바이어슈트라스 M-판정법에 의해 \(\displaystyle f(x)=\sum_{n=0}^{\infty}{\frac{1}{2^{n}}\cos(3^{n}x)}\)에 균등수렴하나 \(\displaystyle f'_{n}(x)=-\sum_{n=1}^{\infty}{\frac{3^{n}}{2^{n}}\sin3^{n}x}\)은 균등수렴하지 않고, \(f\)는 실수 전체에서 미분가능하지 않다.

함수열 \(\displaystyle f_{n}(x)=\frac{\sin nx}{\sqrt{n}}\)은 바이어슈트라스 M-판정법에 의해 \(f(x)=0\)으로 균등수렴하나 \(f_{n}'(x)=\sqrt{n}\cos nx\)이고 \(\displaystyle\lim_{n\,\rightarrow\,\infty}{f_{n}'(x)}=0\)이다.


멱급수 \(\displaystyle f(x)=\sum_{n=0}^{\infty}{a_{n}x^{n}}\)의 수렴반지름이 \(R>0\)이면, \(f\)는 \((-R,\,R)\)에서 미분가능하고 \(\displaystyle f'(x)=\sum_{n=1}^{\infty}{na_{n}x^{n-1}}\,(|x|<R)\)이다. 

증명: \(\displaystyle\lim_{n\,\rightarrow\,\infty}{n^{\frac{1}{n}}}=1\)이므로 \(\displaystyle\lim_{n\,\rightarrow\,\infty}{\sup|na_{n}|^{\frac{1}{n}}}=\lim_{n\,\rightarrow\,\infty}{|a_{n}|^{\frac{1}{n}}}\)이고, 따라서 두 멱급수 \(\displaystyle\sum_{n=0}^{\infty}{a_{n}x^{n}}\)과 \(\displaystyle\sum_{n=1}^{\infty}{na_{n}x^{n-1}}\)의 수렴반지름은 같고, \(\displaystyle\sum_{n=1}^{\infty}{na_{n}x^{n-1}}\)은 임의의 \([a,\,b]\subset(-R,\,R)\)에서 균등수렴한다. 따라서 \(f\)는 \([a,\,b]\)에서 미분가능하고, 모든 \(x\in[a,\,b]\)에 대해 \(\displaystyle f'(x)=\sum_{n=1}^{\infty}{na_{n}x^{n-1}}\)이다. \(x\in[a,\,b]\subset(-R,\,R)\)인 임의의 닫힌구간 \([a,\,b]\)를 선택할 수 있으므로 모든 \(x\in(-R,\,R)\)에서 \(\displaystyle f(x)=\sum_{n=1}^{\infty}{na_{n}x^{n-1}}\)이 성립한다.


참고자료:

Introduction to Mathematical Analysis, Parzynski, Zipse, McGraw-Hill

실해석학 개론, 정동명, 조승제, 경문사

알기쉬운 해석학, 장건수 외 5인, 대선                        

반응형
Posted by skywalker222